Những câu hỏi liên quan
Julian Edward
Xem chi tiết
Hoàng Tử Hà
19 tháng 2 2021 lúc 21:48

\(f\left(-2\right)=-2m+1\)

\(\lim\limits_{x\rightarrow-2^+}f\left(x\right)=\lim\limits_{x\rightarrow-2^+}\dfrac{x^2-3x+2}{x^3+8}=\lim\limits_{x\rightarrow-2^+}\dfrac{\left(x-2\right)\left(x-1\right)}{\left(x+2\right)\left(x^2-2x+4\right)}=\lim\limits_{x\rightarrow-2^+}\dfrac{x-1}{x^2-2x+4}=\dfrac{-2-1}{4-2.\left(-2\right)+4}=-\dfrac{1}{4}\)

\(f\left(-2\right)\ne\lim\limits_{x\rightarrow-2^-}f\left(x\right)\Leftrightarrow-2m+1\ne-\dfrac{1}{4}\Leftrightarrow m\ne\dfrac{5}{8}\)

Bình luận (0)
Dương Nguyễn
Xem chi tiết
Nguyễn Việt Lâm
14 tháng 3 2022 lúc 22:23

a.

\(\lim\limits_{x\rightarrow+\infty}\left(\sqrt{x^2-ax+2021}-x+1\right)\)

\(=\lim\limits_{x\rightarrow+\infty}\left(\dfrac{\left(\sqrt{x^2-ax+2021}-x\right)\left(\sqrt{x^2-ax+2021}+x\right)}{\sqrt{x^2-ax+2021}+x}+1\right)\)

\(=\lim\limits_{x\rightarrow+\infty}\left(\dfrac{-ax+2021}{\sqrt{x^2-ax+2021}+x}+1\right)\)

\(=\lim\limits_{x\rightarrow+\infty}\left(\dfrac{x\left(-a+\dfrac{2021}{x}\right)}{x\left(\sqrt{1-\dfrac{a}{x}+\dfrac{2021}{x^2}}+1\right)}+1\right)\)

\(=\lim\limits_{x\rightarrow+\infty}\left(\dfrac{-a+\dfrac{2021}{x}}{\sqrt{1-\dfrac{a}{x}+\dfrac{2021}{x^2}}+1}+1\right)\)

\(=\dfrac{-a+0}{\sqrt{1+0+0}+1}+1=-\dfrac{a}{2}+1\)

\(\Rightarrow a^2=-\dfrac{a}{2}+1\Rightarrow2a^2+a-2=0\)

Pt trên có 2 nghiệm pb nên có 2 giá trị a thỏa mãn

Bình luận (3)
Nguyễn Việt Lâm
14 tháng 3 2022 lúc 22:24

b.

\(\lim\limits_{x\rightarrow-1}f\left(x\right)=\lim\limits_{x\rightarrow-1}\dfrac{x^3+1}{x+1}\)

\(=\lim\limits_{x\rightarrow-1}\dfrac{\left(x+1\right)\left(x^2-x+1\right)}{x+1}=\lim\limits_{x\rightarrow-1}\left(x^2-x+1\right)\)

\(=1+1+1=3\)

\(f\left(-1\right)=3a\)

Hàm gián đoạn tại điểm \(x_0=-1\) khi:

\(\lim\limits_{x\rightarrow-1}f\left(x\right)\ne f\left(-1\right)\Rightarrow3\ne3a\)

\(\Rightarrow a\ne1\)

Bình luận (0)
Nguyễn Việt Lâm
14 tháng 3 2022 lúc 22:32

c.

Tứ diện ABCD đều \(\Rightarrow\Delta ABD\) đều

\(\widehat{\left(\overrightarrow{DA};BD\right)}=180^0-\widehat{\left(\overrightarrow{DA};\overrightarrow{DB}\right)}=180^0-\widehat{ADB}=180^0-60^0=120^0\)

d.

\(\lim\limits_{x\rightarrow1}f\left(x\right)=\lim\limits_{x\rightarrow1}\dfrac{x^2-1}{2-2x}=\lim\limits_{x\rightarrow1}\dfrac{\left(x-1\right)\left(x+1\right)}{-2\left(x-1\right)}\)

\(=\lim\limits_{x\rightarrow1}\dfrac{x+1}{-2}=\dfrac{1+1}{-2}=-1\)

Để hàm liên tục tại \(x=1\)

\(\Rightarrow f\left(1\right)=\lim\limits_{x\rightarrow1}f\left(x\right)=-1\)

e.

Hàm \(f\left(x\right)\) là hàm đa thức nên liên tục trên R

\(f\left(0\right)=-1< 0\) ; \(f\left(1\right)=2>0\)

\(\Rightarrow f\left(0\right).f\left(1\right)< 0\Rightarrow f\left(x\right)\) luôn có ít nhất 1 nghiệm thuộc \(\left(0;1\right)\)

Do \(\left(0;1\right)\) đồng thời là tập con của \(\left(-1;1\right)\) ; \(\left(-5;3\right)\) và R nên \(f\left(x\right)\) cũng có nghiệm trên các khoảng này

Vậy B là đáp án sai

Bình luận (0)
27. Trần Thanh Nhã 9A3
Xem chi tiết
Nguyễn Việt Lâm
2 tháng 1 lúc 17:34

\(\lim\limits_{x\rightarrow1}f\left(x\right)=\lim\limits_{x\rightarrow1}\dfrac{x^3-x^2+2x-2}{x-1}=\lim\limits_{x\rightarrow1}\dfrac{x^2\left(x-1\right)+2\left(x-1\right)}{x-1}\)

\(=\lim\limits_{x\rightarrow1}\dfrac{\left(x-1\right)\left(x^2+2\right)}{x-1}=\lim\limits_{x\rightarrow1}\left(x^2+2\right)=3\)

\(f\left(1\right)=3.1+m=m+3\)

Hàm số liên tục tại \(x_0=1\) khi và chỉ khi \(\lim\limits_{x\rightarrow1}f\left(x\right)=f\left(1\right)\)

\(\Rightarrow m+3=3\Rightarrow m=0\)

Bình luận (0)
Julian Edward
Xem chi tiết
Hoàng Tử Hà
19 tháng 2 2021 lúc 21:42

\(f\left(0\right)=2.0+m+1=m+1\)

\(\lim\limits_{x\rightarrow0^+}f\left(x\right)=\lim\limits_{x\rightarrow0^+}\dfrac{\sqrt[3]{x+1}-1}{x}=\lim\limits_{x\rightarrow0^+}\dfrac{x+1-1}{x(\sqrt[3]{\left(x+1\right)^2}+\sqrt[3]{x+1}+1)}=\dfrac{1}{1+1+1}=\dfrac{1}{3}\)\(f\left(0\right)=\lim\limits_{x\rightarrow0^+}f\left(x\right)\Leftrightarrow m+1=\dfrac{1}{3}\Rightarrow m=-\dfrac{2}{3}\)

Bình luận (0)
títtt
Xem chi tiết
Nguyễn Lê Phước Thịnh
19 tháng 11 2023 lúc 19:59

Khi \(x\ne1\) thì \(f\left(x\right)=\dfrac{3x^2-3x}{x-1}=\dfrac{3x\left(x-1\right)}{x-1}=3x\) hoàn toàn xác định

nên f(x) liên tục trên các khoảng \(\left(-\infty;1\right);\left(1;+\infty\right)\)(1)

\(\lim\limits_{x\rightarrow1}f\left(x\right)=\lim\limits_{x\rightarrow1}\dfrac{3x^2-3x}{x-1}\)

\(=\lim\limits_{x\rightarrow1}\dfrac{3x\left(x-1\right)}{x-1}=\lim\limits_{x\rightarrow1}3x=3\cdot1=3\)

\(f\left(1\right)=m\cdot1+1=m+1\)

Để hàm số liên tục trên R thì hàm số cần liên tục trên các khoảng sau: \(\left(-\infty;1\right);\left(1;+\infty\right)\) và liên tục luôn tại x=1(2)

Từ (1),(2) suy ra để hàm số liên tục trên R thì hàm số cần liên tục tại x=1

=>\(f\left(1\right)=\lim\limits_{x\rightarrow1}f\left(x\right)\)

=>m+1=3

=>m=2

Bình luận (0)
Đặng Quang Vinh
Xem chi tiết
Hồng Phúc
18 tháng 3 2022 lúc 14:40

\(\dfrac{\sqrt{2x+7}-\sqrt{x+3}-5}{x-1}\) hay \(\dfrac{\sqrt{2x+7}+\sqrt{x+3}-5}{x-1}\)

Bình luận (0)
Julian Edward
Xem chi tiết
Nguyễn Việt Lâm
2 tháng 3 2021 lúc 22:01

\(\lim\limits_{x\rightarrow1^+}f\left(x\right)=\lim\limits_{x\rightarrow1^+}\dfrac{\sqrt{x+3}-2}{x-1}=\lim\limits_{x\rightarrow1^+}\dfrac{x-1}{\left(x-1\right)\left(\sqrt{x+3}+2\right)}=\lim\limits_{x\rightarrow1^+}\dfrac{1}{\sqrt{x+3}+2}=\dfrac{1}{4}\)

\(f\left(1\right)=\lim\limits_{x\rightarrow1^-}f\left(x\right)=\lim\limits_{x\rightarrow1^-}\left(mx\right)=m\)

Hàm liên tục tại x=1 khi: \(\lim\limits_{x\rightarrow1^+}f\left(x\right)=\lim\limits_{x\rightarrow1^-}f\left(x\right)=f\left(1\right)\)

\(\Leftrightarrow m=\dfrac{1}{4}\)

Bình luận (0)
títtt
Xem chi tiết
Nguyễn Lê Phước Thịnh
19 tháng 11 2023 lúc 19:56

Khi \(x\ne-2\) thì \(f\left(x\right)=\dfrac{3x^2+5x-2}{x+2}\) là một hàm phân thức hoàn toàn xác định nên f(x) liên tục tại các khoảng \(\left(-\infty;-2\right);\left(-2;+\infty\right)\)(1)

\(\lim\limits_{x\rightarrow-2}f\left(x\right)=\lim\limits_{x\rightarrow-2}\dfrac{3x^2+5x-2}{x+2}\)

\(=\lim\limits_{x\rightarrow-2}\dfrac{3x^2+6x-x-2}{x+2}\)

\(=\lim\limits_{x\rightarrow-2}\dfrac{\left(x+2\right)\left(3x-1\right)}{x+2}\)

\(=\lim\limits_{x\rightarrow-2}3x-1=3\cdot\left(-2\right)-1=-7\)

\(f\left(-2\right)=m\)

Để hàm số liên tục trên R thì hàm số liên tục tại x=2 và liên tục tại các khoảng \(\left(-\infty;-2\right);\left(-2;+\infty\right)\)(2)

Từ (1),(2) suy ra Để hàm số liên tục trên R thì hàm số liên tục tại x=2

=>\(\lim\limits_{x\rightarrow-2}f\left(x\right)=f\left(-2\right)\)

=>m=-7

Bình luận (0)
títtt
Xem chi tiết
Nguyễn Lê Phước Thịnh
19 tháng 11 2023 lúc 19:32

\(\lim\limits_{x\rightarrow1}f\left(x\right)=\lim\limits_{x\rightarrow1}\dfrac{2x^2-5x+3}{x-1}\)

\(=\lim\limits_{x\rightarrow1}\dfrac{\left(x-1\right)\left(2x-3\right)}{x-1}=\lim\limits_{x\rightarrow1}2x-3=2\cdot1-3=-1\)

f(1)=4

=>\(\lim\limits_{x\rightarrow1}f\left(x\right)< >f\left(1\right)\)

=>Hàm số bị gián đoạn tại x=1

 

Bình luận (0)